Re: [obm-l] Problema sobre complexos [ime 2003-2004]

2004-10-21 Por tôpico Marcio Cohen
pude notar que a banca retirou, na primeira correção, 60% da questão por esse erro. []s Marcio - Original Message - From: Felipe Torres [EMAIL PROTECTED] To: [EMAIL PROTECTED] Sent: Friday, October 22, 2004 12:21 AM Subject: [obm-l] Problema sobre complexos [ime 2003-2004] Oi. eu

Re: [obm-l] Problema Legal

2004-10-16 Por tôpico Claudio Buffara
Tm toda a razao. Eu esqueci do 96 = 3*32. Obrigado pela correcao. []s, Claudio. on 16.10.04 00:37, Eduardo Henrique Leitner at [EMAIL PROTECTED] wrote: o 96 tb seria azul nao? seguindo sua lohgica: 3 6 12 24 48 96, cujo maior divisor impar eh 3: 4*0 + 3 entao teremos 48 azuis e

Re: [obm-l] Problema Legal

2004-10-15 Por tôpico Eduardo Henrique Leitner
o 96 tb seria azul nao? seguindo sua lohgica: 3 6 12 24 48 96, cujo maior divisor impar eh 3: 4*0 + 3 entao teremos 48 azuis e portando 52 vermelhos, correto? On Wed, Oct 13, 2004 at 10:19:59AM -0200, Claudio Buffara wrote: on 12.10.04 18:09, benedito at [EMAIL PROTECTED] wrote: Abaixo,

Re: [obm-l] Problema Legal

2004-10-13 Por tôpico Claudio Buffara
on 12.10.04 18:09, benedito at [EMAIL PROTECTED] wrote: Abaixo, segue um problema legal: Problema Num corredor, existem 100 armários em fila, numeradas de 1 até 100. Um pintor vem e pinta todas os armários de vermelho. Em seguida, vem um segundo pintor e pinta de azul os

[obm-l] Problema Legal

2004-10-12 Por tôpico benedito
Abaixo, segue um problema legal: Problema Num corredor, existem 100 armários em fila, numeradas de 1 até 100. Um pintor vem e pinta todas os armários de vermelho. Em seguida, vem um segundo pintor e pinta de azul os armários de três em três, começando do armário número 3. A seguir, vem

[obm-l] Problema 1 - Ibero 2004

2004-10-01 Por tôpico Marcelo Ribeiro
Alguém fez o problema 1 da Ibero 2004? Realmente gostaria de ver a solução. agradeço abraço Marcelo Yahoo! Acesso Grátis - Internet rápida e grátis. Instale o discador agora!

Re:[obm-l] Problema-HELP

2004-09-29 Por tôpico Osvaldo Mello Sponquiado
É possível encontrar uma função de duas varíaveis Z=F(X,Y) cujo o gráfico é uma reta em R³? Suponha que F esteja definida em um subconjunto aberto e conexo de RxR. Como F é uma função a cada par (x,y) está associado um único z(x,y), assim a única maneira de obter uma reta como sendo um

Re: [obm-l] Problema-HELP

2004-09-29 Por tôpico Artur Costa Steiner
: [obm-l] Problema-HELP Data: 29/09/04 05:27 lá vai um probleminha que o professor Cleto de França da Universidade de PE mandou enviar: É possível encontrar uma função de duas varíaveis Z=F(X,Y) cujo o gráfico é uma reta em R³? Aluno desperado da POLI Valério Badarau

[obm-l] Problema-HELP

2004-09-28 Por tôpico valeriomoura
lá vai um probleminha que o professor Cleto de França da Universidade de PE mandou enviar: É possível encontrar uma função de duas varíaveis Z=F(X,Y) cujo o gráfico é uma reta em R³? Aluno desperado da POLI Valério Badarau

[obm-l] Problema 4 do nível u.

2004-09-19 Por tôpico Fernando v
Olá a todos. Na solução apresantada no site da obm para a questão 4 do nível u (aquela dos subconjuntos), é usado um raciocínio em que o total de casos é 7^n e se exclui os casos que não servem, na prova o meu raciocínio era que o total de casos era 2^3n e também eu exclui os casos que não

[obm-l] Problema - Nivel 2 - Olimpíada Portuguesa de Matemática

2004-09-15 Por tôpico Daniel S. Braz
Pessoal, Dispomos de uma mesa de bilhar de 8 metros de comprimento e 2 metros de largura com uma única bola no seu centro. A bola é lançada em linha recta e, depois de percorrer 29 metros, pára numa esquina da mesa. Quantas vezes tocou a bola nos bordos da mesa? Nota: Quando a bola toca num bordo

[obm-l] Re:[obm-l] Problema envolvendo potências

2004-09-10 Por tôpico eritotutor
Amigo Rafael, Eh a primeira vez que respondo se estiver errado, me corrijam... Divida o numerador e o denominador da fraçao por 3^29 e vc obterah o seguinte: {(9 + 4(2/3)^29}:{1 + (2/3)^29 Analisando esse resultado segue q (2/3)^29 e um numero menor do que 1. Daí {(9 + 4(2/3)^29}:{1 +

Re: [obm-l] Re:[obm-l] Problema envolvendo potências

2004-09-10 Por tôpico Bernardo Freitas Paulo da Costa
Bom, a idéia que você teve está quase certa, mas você deslizou na hora de fazer a divisão (pois aí o sinal da desigualdade muda). Temos, como você falou, x = (3^31 + 2^31)/(3^29 + 2^29) = (9 + 4y)/(1 + y), onde y = (2/3)^29 Podemos escrever 9 + 4y = 9 + 9y - 5y = 9(1 + y) - 5y, e

Re: [obm-l] Re:[obm-l] Problema envolvendo potências

2004-09-10 Por tôpico kleinad
Seja k = (9*3^29 + 4*2^29)/(3^29 + 2^29). Repare que 9 = (9*3^29 + 9*2^29)/ (2^29 + 3^29) k. Resta mostrar que k 8. Basta mostrar que 9*3^29 + 4*2^29 - 8*3^29 - 8*2^29 = 3^29 - 4*2^29 0. Temos 3^29 = (1 + 2)^29 2^29 + 29*2^28 2^29 + 14*2^29 = 15*2^29, donde 3^29 - 4*2^29 15*2^29 - 4*2^29 =

[obm-l] Problema envolvendo potências

2004-09-09 Por tôpico Rafael Silva
Olá pessoal! Por favor resolvam esse problema para mim (é um pouco pesado para um estudante de 6ª série) Qual o maior inteiro maior ou igual a (331 +231 )/(329+229 ) ?Abraços Rafael Silva E-mail:[EMAIL PROTECTED] Yahoo! Acesso Grátis - navegue de graça com conexão de

[obm-l] Re:[obm-l] PROBLEMA DE OLIMPÍADA!

2004-09-07 Por tôpico Paulo Jose Rodrigues
Este problema não é da OCM! Este problema, publicado na coluna do Jornal O Povo como treinamento para a OCM é da AIME-- equivalente a segunda fase da olimpíada norte-americana. Oi, Pessoal! Uma faixa de papel tem 1024 unidades de comprimento e 1 unidade de largura dividida em 1024

[obm-l] PROBLEMA DE OLIMPÍADA!

2004-09-06 Por tôpico jorgeluis
Oi, Pessoal! Uma faixa de papel tem 1024 unidades de comprimento e 1 unidade de largura dividida em 1024 quadrados unitários. A faixa é dobrada 8 vezes pelas pontas de modo a formar uma pilha de 1024 quadrados unitários. Quantos destes quadrados estão abaixo do quadrado que era originalmente 942º

RE: [obm-l] Problema das Oito Rainhas...

2004-08-30 Por tôpico Qwert Smith
Alguem poderia me ajudar com a solução desse problema Estou realmente precisando... Obrigrado... se for possivel me mandar a solução para esse e-mail ou [EMAIL PROTECTED] ficaria muito grato! Problema - colocar oito rainhas no tabuleiro de xadrez modo que nenhuma delas ataque as outras.

Re: [obm-l] Problema das Oito Rainhas...

2004-08-30 Por tôpico Augusto Cesar de Oliveira Morgado
) 2295-2978 Empresa 100% Brasileira - Desde 1992 prestando servicos online -- Original Message --- From: Rodrigo Alberto [EMAIL PROTECTED] To: [EMAIL PROTECTED] Sent: Mon, 30 Aug 2004 14:31:58 -0300 (ART) Subject: [obm-l] Problema das Oito Rainhas... Alguem poderia me ajudar

Re: [obm-l] Problema das Oito Rainhas...

2004-08-30 Por tôpico Domingos Jr.
Que tal deixar de ser preguiçoso e pesquisar algum livro de IA como o AIMA (artifficial intelligence - a modern approach). Alguem poderia me ajudar com a solução desse problema Estou realmente precisando... Obrigrado... se for possivel me mandar a solução para esse e-mail ou [EMAIL

RE: [obm-l] Problema das Oito Rainhas...

2004-08-30 Por tôpico Johann Peter Gustav Lejeune Dirichlet
Nossa, realmente... Esse e um problema quase que puramente computacional. Ainda mais pelo"Use Heuristicas".Pede pro Qwert escrever um programinha, oras! Ou tente sozinho. Mas se a preguiça for maior, veja http://www.icmc.usp.br/coteia/mostra.php?ident=13.3 e va na segunda aula de recursividade.

[obm-l] Problema do pica-pau

2004-08-19 Por tôpico Rafael Silva
Olá gente! Queria que vocês resolvessem esse problema para mim (nem que pareça besta, é apenas para confirmar meu resultado). Um pica-pau marca a bicadas seu caminho em uma árvore, realizando seu caminho em espiral (como se fosse a hélice de um avião) e começando 20 metros acima do solo. Sabendo

[obm-l] Problema 6, IMO 2004 (Atenas, Grecia)

2004-08-06 Por tôpico Johann Peter Gustav Lejeune Dirichlet
6-Um numero natural e dito alternante se dois digitos consecutivos sao de paridades diferentes. (Como um exemplo, temos 1, 2, 12567825650167.) Determine todos os numeros naturais que nao dividem nenhum numero alternante. Resposta: todos os multiplos de 20 e apenas esses . Vamos dizer que x e

[obm-l] Problema De Engenharia

2004-07-25 Por tôpico Welma Pereira
Olá amigos, gostaria de pedir uma ajudinha num problema para os engenheiros da lista vou escrever em ingles para ter certeza de que nao perca o real significado A digital filter has transfer function G( z)= ( 6z ( 5z+1) ) / ( ( 2z - 1 ) ( 3z +2)

Re: RES: [obm-l] Problema Subconjuntos

2004-07-22 Por tôpico Poncio
, P.C.Pinto Carvalho e Pedro Fernandez, da coleção do Professor de Matemática. Espero ter ajudado,um grande abraço, Poncio - Original Message - From: Artur Costa Steiner [EMAIL PROTECTED] To: [EMAIL PROTECTED] Sent: Wednesday, July 21, 2004 8:14 PM Subject: Re: RES: [obm-l] Problema Subconjuntos

Re: [obm-l] Problema Subconjuntos. Correção

2004-07-21 Por tôpico Augusto Cesar de Oliveira Morgado
Favor esquecer a bobagem abaixo. Morgado -- Original Message --- From: Augusto Cesar de Oliveira Morgado [EMAIL PROTECTED] To: [EMAIL PROTECTED] Sent: Wed, 21 Jul 2004 02:51:09 -0200 Subject: Re: RES: [obm-l] Problema Subconjuntos C(n-2;3). Basta usar o primeiro lema de

Re: [obm-l] Problema - Matemática Discreta

2004-07-21 Por tôpico Domingos Jr.
Se existe uma pessoa com pelo menos n conhecidos, nada temos a provar. Se não, escolha uma pessoa qualquer: ela conhece no máximo n-1 pessoas. Elimine ela e os conhecidos e fique com = (m-2)n + 1 pessoas, repita o passo m-1 vezes e você terá obtido um conjunto de m pessoas que não se conhecem.

Re: RES: [obm-l] Problema Subconjuntos

2004-07-21 Por tôpico Helder Suzuki
achei isso no arquivo da lista: quote Kaplansky. Primeiro lema: O número de subconjuntos de tamanho p do conjunto {1, 2,..., n} no qual nao figuram numeros consecutivos eh C(n-p+1, p) Segundo lema: Igual ao anterior, mas considerando 1 e n como consecutivos. O numero de subconjuntos eh

[obm-l] Problema Subconjuntos

2004-07-20 Por tôpico David M. Cardoso
Olá, Alguem pode me ajudar? Não consegui resolver o seguinte problema: Quantos subconjuntos o conjunto {1,2,3,...,n} tais que não contêm três inteiros consecutivos? A dica dada na questão é: Encontre uma recorrência. Porém, qualquer solução (sem/com recorrência) vai ajudar. []'s David

[obm-l] Problema de Divisibilidade / Primos

2004-07-20 Por tôpico David M. Cardoso
Mais duas questoes que não consigo me mecher: Quantos inteiros existem que não são divisíveis por qualquer que seja o primo maior que 20 e não são divisiveis por qualquer que seja o primo? []'s David = Instruções para

Re: [obm-l] Problema de Divisibilidade / Primos

2004-07-20 Por tôpico Bruno França dos Reis
-BEGIN PGP SIGNED MESSAGE- Hash: SHA1 On Tuesday 20 July 2004 18:26, David M. Cardoso wrote: Mais duas questoes que não consigo me mecher: Quantos inteiros existem que não são divisíveis por qualquer que seja o primo maior que 20 e não são divisiveis por qualquer que seja o primo? a)

Re: [obm-l] Problema de Divisibilidade / Primos

2004-07-20 Por tôpico Alessandro
h, agora vc me deixou com uma duvida, pois ateh hj sabia q o numero 1 era primos, mas nao era considerado como primo por ser composto,( o mesmo acontecia com o 2, ou estou ficando loko ;) On Tuesday 20 July 2004 18:53, Bruno França dos Reis wrote: ] On Tuesday 20 July 2004 18:26, David

Re: [obm-l] Problema Subconjuntos

2004-07-20 Por tôpico Helder Suzuki
vamos ver, seguindo a dica de usar recorrencia se T[n] for igual ao numero de subconjuntos do conjunto {1, 2, ..., n} que nao contem 3 inteiros consecutivos. temos que: T[0] = 1 {} T[1] = 2 {} e {1} T[2] = 4 {}, {1}, {2} e {1, 2} T[3] = 7 {}, {1}, {2}, {1, 2}, {3}, {1, 3}, {2, 3} T[4] = 13

Re: [obm-l] Problema de Divisibilidade / Primos

2004-07-20 Por tôpico Bruno França dos Reis
-BEGIN PGP SIGNED MESSAGE- Hash: SHA1 On Tuesday 20 July 2004 19:20, Alessandro wrote: h, agora vc me deixou com uma duvida, pois ateh hj sabia q o numero 1 era primos, mas nao era considerado como primo por ser composto,( o mesmo acontecia com o 2, ou estou ficando loko ;) 1 é

RES: [obm-l] Problema de Divisibilidade / Primos

2004-07-20 Por tôpico David M. Cardoso
18:53 Para: [EMAIL PROTECTED] Assunto: Re: [obm-l] Problema de Divisibilidade / Primos -BEGIN PGP SIGNED MESSAGE- Hash: SHA1 On Tuesday 20 July 2004 18:26, David M. Cardoso wrote: Mais duas questoes que não consigo me mecher: Quantos inteiros existem que não são divisíveis

Re: RES: [obm-l] Problema de Divisibilidade / Primos

2004-07-20 Por tôpico Bernardo Freitas Paulo da Costa
: terça-feira, 20 de julho de 2004 18:53 Para: [EMAIL PROTECTED] Assunto: Re: [obm-l] Problema de Divisibilidade / Primos -BEGIN PGP SIGNED MESSAGE- Hash: SHA1 On Tuesday 20 July 2004 18:26, David M. Cardoso wrote: Mais duas questoes que não consigo me mecher: Quantos

RES: [obm-l] Problema de Divisibilidade / Primos

2004-07-20 Por tôpico David M. Cardoso
... menos o vazio.. temos entao 2^8 - 1 numeros deste tipo. Ta certo? []'s David -Mensagem original- De: [EMAIL PROTECTED] [mailto:[EMAIL PROTECTED] Em nome de David M. Cardoso Enviada em: terça-feira, 20 de julho de 2004 20:11 Para: [EMAIL PROTECTED] Assunto: RES: [obm-l] Problema de

RES: [obm-l] Problema Subconjuntos

2004-07-20 Por tôpico David M. Cardoso
PROTECTED] Em nome de Helder Suzuki Enviada em: terça-feira, 20 de julho de 2004 19:30 Para: [EMAIL PROTECTED] Assunto: Re: [obm-l] Problema Subconjuntos vamos ver, seguindo a dica de usar recorrencia se T[n] for igual ao numero de subconjuntos do conjunto {1, 2, ..., n} que nao contem 3

RES: RES: [obm-l] Problema de Divisibilidade / Primos

2004-07-20 Por tôpico David M. Cardoso
: Re: RES: [obm-l] Problema de Divisibilidade / Primos Oi, David, Enumere os primos menores do que 20: 2, 3, 5, 7, 11, 13, 17, 19: são 8. Um número que satisfaça as condições do enunciado pode ter, no máximo, um de cada um destes fatores, pela segunda parte, e nenhum outro fator, pela

[obm-l] Problema - Recorrência / Fibonacci

2004-07-20 Por tôpico David M. Cardoso
Olá novamente, Seja F_n a recorrência definida por F_(n+1) = F_n + F_(n-1). Com F_1 = 1, F_2 = 1, ... (sequencia de fibonacci) Qual é o maior: 2^100 ou F_100 ? deu pra perceber, testando, que 2^100 é maior. Ateh porque 2^(n+1) / 2^n = 2 Enquanto que F_(n+1) / F_(n) ~ 1,618 quando n é grande.

RES: [obm-l] Problema - Primos

2004-07-20 Por tôpico David M. Cardoso
Mostre que um número com 30 dígitos não pode ter mais que 100 fatores primos. Bem.. talvez eu tenha feito, acho que eh soh mostrar que Piso[Log_10[2^100]+1] = 31 e que portanto 2^100, que é o menor produto de 100 fatores primos, tem 31 dígitos. []'s David

Re: [obm-l] Problema - Primos

2004-07-20 Por tôpico Domingos Jr.
David M. Cardoso wrote: Mais um problema não resolvido: Mostre que um número com 30 dígitos não pode ter mais que 100 fatores primos. o menor número com 100 fatores primos é p_1 * p_2 * ... * p_100 onde p_1, p_2, .. p_100 são os 100 primeiros primos note que 2, 3, 5, 7 são os únicos primos menores

Re: [obm-l] Problema - Recorrência / Fibonacci

2004-07-20 Por tôpico Domingos Jr.
David M. Cardoso wrote: Olá novamente, Seja F_n a recorrência definida por F_(n+1) = F_n + F_(n-1). Com F_1 = 1, F_2 = 1, ... (sequencia de fibonacci) Qual é o maior: 2^100 ou F_100 ? deu pra perceber, testando, que 2^100 é maior. Ateh porque 2^(n+1) / 2^n = 2 Enquanto que F_(n+1) / F_(n) ~ 1,618

[obm-l] Problema - Matemática Discreta

2004-07-20 Por tôpico David M. Cardoso
Eu não sei em que tópico este problema se enquadra, por isso coloquei no assunto a disciplina que tem relação com ele. Não consegui fazer: Existem (m-1)n + 1 pessoas na sala. Mostre que ou existem m pessoas que não se conhecem mutuamente, ou existe uma pessoa que conhece pelo menos n outras.

[obm-l] RES: [obm-l] Problema - Recorrência / Fibonacci

2004-07-20 Por tôpico David M. Cardoso
Entendi.. entendi.. obrigado. []'s -Mensagem original- De: [EMAIL PROTECTED] [mailto:[EMAIL PROTECTED] Em nome de Domingos Jr. Enviada em: terça-feira, 20 de julho de 2004 23:44 Para: [EMAIL PROTECTED] Assunto: Re: [obm-l] Problema - Recorrência / Fibonacci David M. Cardoso

Re: RES: [obm-l] Problema Subconjuntos

2004-07-20 Por tôpico Augusto Cesar de Oliveira Morgado
-2978 Empresa 100% Brasileira - Desde 1992 prestando servicos online -- Original Message --- From: David M. Cardoso [EMAIL PROTECTED] To: [EMAIL PROTECTED] Sent: Tue, 20 Jul 2004 20:57:24 -0300 Subject: RES: [obm-l] Problema Subconjuntos Cara, muito obrigado.. Sendo que ta dando

Re: [obm-l] Problema Subconjuntos

2004-07-20 Por tôpico Claudio Buffara
Oi, Helder: Eu achei uma recorrencia diferente: Seja A um dos T(n) subconjuntos nas condicoes do enunciado. Existem 3 casos a considerar: Caso 1: n nao pertence a A == existem T(n-1) tais subconjuntos Caso 2: n pertence mas n-1 nao pertence a A == existem T(n-2) tais subconjuntos Caso 3: n

Re: RES: [obm-l] Problema Subconjuntos

2004-07-20 Por tôpico Helder Suzuki
completa o suficiente.. []'s David -Mensagem original- De: [EMAIL PROTECTED] [mailto:[EMAIL PROTECTED] Em nome de Helder Suzuki Enviada em: terça-feira, 20 de julho de 2004 19:30 Para: [EMAIL PROTECTED] Assunto: Re: [obm-l] Problema Subconjuntos vamos ver, seguindo

[obm-l] Problema em semi-aberto

2004-07-16 Por tôpico Faelccmm
Ola pessoal, Aqui na lista foi dada uma solucao por equacoes de recorrencia --por isso disse semi-aberto no thread -- mas acredito que haja uma solucao atraves de matematica de Ensino medio -- por combinatoria, talvez. Pois este problema caiu na OBM de 1997 (fase senior) . Alguem se propoe a

Re: [obm-l] Problema em semi-aberto

2004-07-16 Por tôpico Augusto Cesar de Oliveira Morgado
) 2542-4849, (21) 2295-3331        Fax: (21) 2295-2978 Empresa 100% Brasileira - Desde 1992 prestando servicos online -- Original Message --- From: [EMAIL PROTECTED] To: [EMAIL PROTECTED] Sent: Fri, 16 Jul 2004 02:44:17 EDT Subject: [obm-l] Problema em semi-aberto Ola

Re: [obm-l] Problema em semi-aberto

2004-07-16 Por tôpico Faelccmm
://www.centroin.com.br Tel: (21) 2542-4849, (21) 2295-3331 Fax: (21) 2295-2978 Empresa 100% Brasileira - Desde 1992 prestando servicos online -- Original Message --- From: [EMAIL PROTECTED] To: [EMAIL PROTECTED] Sent: Fri, 16 Jul 2004 02:44:17 EDT Subject: [obm-l] Problema em semi-aberto

Re:[obm-l] Problema interessante de PA

2004-07-10 Por tôpico Fernando Villar
participação! [ ],s Fernando - Original Message - From: claudio.buffara To: obm-l Sent: Saturday, July 03, 2004 10:00 AM Subject: Re:[obm-l] Problema interessante de PA Eh sim. 0 = 0 + 0. O enunciado nao fala nada sobre cada termo ser a soma de termos difere

RE: [obm-l] Problema

2004-07-08 Por tôpico Qwert Smith
Imagine um quadrado ABCD de lado a. Imagine agora dois móveis partindo do mesmo ponto A. O móvel 1 faz os caminhos AB, BC, CD , DA , AB. O móvel 2 percorre os caminhos AC, CA, AC, ou seja, só se move pela diagonal. Com base nisso responda: a) Se esses móveis tiverem mesma velocidade e

[obm-l] problema para pensar

2004-07-08 Por tôpico niski
Pessoal, achei esse problema legal e então estou repassando pra lista. Se alguem quiser a solucao avise. O problema pede que usando uma extensao da desigualdade isoperimetrica, isto é V/[((3/4)*pi)*(A/4pi)^3/2] = 1 Onde V é o volume de um solido qualquer e A é a sua area de superficie, prove

[obm-l] Problema

2004-07-07 Por tôpico Korshinoi
Imagine um quadrado ABCD de lado a. Imagine agora dois móveis partindo do mesmo ponto A. O móvel 1 faz os caminhos AB, BC, CD , DA , AB. O móvel 2 percorre os caminhos AC, CA, AC, ou seja, só se move pela diagonal. Com base nisso responda: a) Se esses móveis tiverem mesma velocidade e

Re:[obm-l] Problema interessante de PA

2004-07-03 Por tôpico Rogerio Ponce
Olá Fernando, usando o que vc mesmo disse anteriormente: (-r,0,r,2r,...) satisfaz a condição mas o primeiro termo não é a soma de dois termos desta mesma PA. Abraços, Rogério. From: f_villar Acho que a condição necessária e suficiente é: um dos termos é o simétrico da razão da PA: Ida: Se um

Re:[obm-l] Problema interessante de PA

2004-07-03 Por tôpico claudio.buffara
. De: [EMAIL PROTECTED] Para: "obm-l" [EMAIL PROTECTED] Cópia: Data: Fri, 2 Jul 2004 19:22:02 -0300 Assunto: Re:[obm-l] Problema interessante de PA Olá Cláudio, tudo bem? Acho que a condição não é suficiente pois considerando a PA: (0, r, 2r,3r,...) 0 per

[obm-l] Problema da OBM 2003

2004-07-03 Por tôpico MatheusHidalgo
Há N cidades em Tumbólia. Cada duas cidades desse país são ligadas por uma rodovia ou uma ferrovia, não existin do nenhum par de cidades ligadas por ambos os meios. Um turista deseja viajar por toda a Tumbólia, visitando cada cidade exatamente uma vez, e retornar a cidade onde ele começou sua

[obm-l] Problema interessante de PA

2004-07-02 Por tôpico f_villar
"Encontrar a condição necessária e suficiente que deve ser verificada para que qualquer termo de uma progressão aritmética infinita seja a soma de dois termos, da mesma progressão. "

RE: [obm-l] Problema interessante de PA

2004-07-02 Por tôpico Rogerio Ponce
É dito que a soma de dois termos da progressão é igual ao dobro de um dos termos mais uma quantidade inteira de vezes a razão da progressão. Por outro lado, em qualquer progressão, isso deve também ser igual a um dos termos mais uma quantidade inteira de vezes a razão da progressão. Portanto, um

Re:[obm-l] Problema interessante de PA

2004-07-02 Por tôpico claudio.buffara
pertence à PA e é positivo == contradição, pois a é o menor termo não-negativo da PA == r = a == 0 = a - r pertence à PA. []s, Claudio. De: [EMAIL PROTECTED] Para: "obm-l" [EMAIL PROTECTED] Cópia: Data: Fri, 2 Jul 2004 15:20:43 -0300 Assunto: [obm-l] Problema inter

Re:[obm-l] Problema interessante de PA

2004-07-02 Por tôpico f_villar
ri, 02 Jul 2004 16:23:48 -0300 Assunto: Re:[obm-l] Problema interessante de PA Condição necessária e suficiente: 0 pertence à PA. Se 0 pertence à PA, então, de duas uma: a PA é constante (razão = 0) ou a razão será igual ao menor termo positivo. Em todo caso, os termos da PA serão

RE: [obm-l] Problema interessante de PA

2004-07-02 Por tôpico f_villar
: [EMAIL PROTECTED] Cópia: Data: Fri, 02 Jul 2004 19:20:18 + Assunto: RE: [obm-l] Problema interessante de PA É dito que a soma de dois termos da progressão é igual ao dobro de um dos termos mais uma quantidade inteira de vezes a razão da progressão. Por outro lado, em

RE: [obm-l] Problema interessante de PA

2004-07-02 Por tôpico Rogerio Ponce
Olá Fernando, sim, sou do Rio! Bem, eu havia imaginado uma sequência infinita nas duas direções. Se existe um primeiro termo, que também deva ser obtido pela soma de 2 outros termos da PA, então, pela minha conclusão anterior, todos os termos são nulos e a razão também é zero. Abraços, Rogério.

Re:[obm-l] Problema interessante de PA

2004-07-02 Por tôpico f_villar
ri, 02 Jul 2004 20:36:40 -0300 Assunto: Re:[obm-l] Problema interessante de PA Acho que a condição necessária e suficiente é: um dos termos é o simétrico da razão da PA: Ida:Se um dos termos é o simétrico da razão então 0 pertence a PA e a razão também é um de seus termos. Podemos divid

Re:[obm-l] Problema da Eureka-retorno

2004-06-28 Por tôpico claudio.buffara
Jun 2004 00:04:24 EDT Assunto: [obm-l] Problema da Eureka-retorno Ola Claudio e demais colegas, Estou retornando a mensagem, pois andei pensando: - Sera que o problema nao admite outra solucao ? Pois equacoes de recorrencia eh um assunto tratado na Eureka 09 e o problema enviado por mim

[obm-l] PROBLEMA DE ALGIBEIRA!

2004-06-28 Por tôpico jorgeluis
Meus Colegas! Fomos presenteados mais uma vez pelo raciocínio do nosso Comandante à respeito da inexistência da lei de conservação dos volumes, pois para se ter idéia do tamanho da brincadeira, este pequeno detalhe foi abordado uma única vez em meados de 80 pelo amigo postal, Prof. Ubiratan

Re: [obm-l] PROBLEMA DE ALGIBEIRA!

2004-06-28 Por tôpico Bruno França dos Reis
-BEGIN PGP SIGNED MESSAGE- Hash: SHA1 On Monday 28 June 2004 19:18, [EMAIL PROTECTED] wrote: A propósito, alguém poderia responder qual a medida de melhor qualidade: 99 cm ou 999 cm? Não seria 999 cm, já que esta tem 3 algarismos significativos, sendo 2 certos e um duvidoso, enquanto

Re: [obm-l] PROBLEMA DE ALGIBEIRA!

2004-06-28 Por tôpico Osvaldo
Eu acredito que dependa unicamente do valor da precisão do instrumento com que se medirão estas grandezas. Falou -BEGIN PGP SIGNED MESSAGE- Hash: SHA1 On Monday 28 June 2004 19:18, [EMAIL PROTECTED] wrote: A propósito, alguém poderia responder qual a medida de melhor

[obm-l] Problema da Eureka-retorno

2004-06-27 Por tôpico Faelccmm
Ola Claudio e demais colegas, Estou retornando a mensagem, pois andei pensando: - Sera que o problema nao admite outra solucao ? Pois equacoes de recorrencia eh um assunto tratado na Eureka 09 e o problema enviado por mim esta na Eureka 01. Creio que os responsaveis por esta revista tratam-na de

[obm-l] problema combinatoria

2004-06-23 Por tôpico rafaelc\.l
De quantos modos 720 pode ser dividido em um produto de três inteiros positivos? __ Acabe com aquelas janelinhas que pulam na sua tela. AntiPop-up UOL - É grátis! http://antipopup.uol.com.br/

Re: [obm-l] problema combinatoria

2004-06-23 Por tôpico Fábio Dias Moreira
-BEGIN PGP SIGNED MESSAGE- Hash: SHA1 rafaelc\.l [EMAIL PROTECTED] said: De quantos modos 720 pode ser dividido em um produto de três inteiros positivos? [...] Como 720 = 2^4*3^2*5, a*b*c = 720 == a = 2^a1*3^a2*5^a3 b = 2^b1*3^b2*5^b3 c = 2^c1*3^c2*5^c3 com a1+b1+c1 = 4, a2+b2+c2 =

Re: [obm-l] problema combinatoria

2004-06-23 Por tôpico Bruno França dos Reis
-BEGIN PGP SIGNED MESSAGE- Hash: SHA1 On Wednesday 23 June 2004 11:25, rafaelc.l wrote: De quantos modos 720 pode ser dividido em um produto de três inteiros positivos? essa é bonitinha. Vc faz assim: fatora o número em questao: 720=10*72 = 2*5*2*36 = 2*5*2*6*6 = 2*5*2*2*3*2*3 =

Re: [obm-l] problema combinatoria

2004-06-23 Por tôpico Bruno França dos Reis
-BEGIN PGP SIGNED MESSAGE- Hash: SHA1 On Wednesday 23 June 2004 12:49, Fábio Dias Moreira wrote: rafaelc\.l [EMAIL PROTECTED] said: De quantos modos 720 pode ser dividido em um produto de três inteiros positivos? [...] Como 720 = 2^4*3^2*5, a*b*c = 720 == a = 2^a1*3^a2*5^a3 b

Re: [obm-l] problema combinatoria

2004-06-23 Por tôpico claudio.buffara
PROTECTED] Cópia: Data: Wed, 23 Jun 2004 12:49:38 -0300 Assunto: Re: [obm-l] problema combinatoria -BEGIN PGP SIGNED MESSAGE- Hash: SHA1 "rafaelc\.l" <[EMAIL PROTECTED]>said: De quantos modos 720 pode ser dividido em um produto de três inteiros positi

[obm-l] problema com limites [u]

2004-06-17 Por tôpico André Martin Timpanaro
Notacões: S_a_b f(x)dx = integral definida de f(x) no intervalo [a,b]. Sum_i=m_n a_i = somatória dos a_i com i variando de m a n. f '(x) = df/dx ___ S_a_b f(x) dx = lim n--+inf (Sum_i=0_n f(i(b-a)/n + a))/n f '(x) = lim n--0 (f(x+n) - f(x))/n

[obm-l] Problema das pesagens

2004-06-17 Por tôpico Nicolau C. Saldanha
On Wed, Jun 16, 2004 at 08:46:07PM -0300, Guilherme Carlos Moreira e Silva wrote: tem um jeito de descobrir se o lado que sobe que tem uma bola mais leve das demais ou se a que desce que tem uma mais pesada? Maurizio [EMAIL PROTECTED] wrote: Pese 3 de cada lado e deixe 6 de lado Se

[obm-l] Problema envolvendo sistema linear

2004-06-15 Por tôpico Maurizio
Em uma cesta de frutas, há 3 vezes mais peras doq ue laranjas. Eu e meus amigos vamos dividir as frutas. Se cada um de nós receber 5 laranjas e 8 peras, restarão 21 peras, e as laranjas serão todas distribuídas. Quantas laranjas há na cesta? Quantas pessoas somos?

RES: [obm-l] Problema envolvendo sistema linear

2004-06-15 Por tôpico Wellington
L = nº laranjas P = nº peras X = nº pessoas Faça: 3L = P 5X = L 8X + 21 = P -Mensagem original- De: [EMAIL PROTECTED] [mailto:[EMAIL PROTECTED] Em nome de Maurizio Enviada em: Tuesday, June 15, 2004 7:17 PM Para: [EMAIL PROTECTED] Assunto: [obm-l] Problema envolvendo sistema linear Em

RES: [obm-l] Problema envolvendo sistema linear

2004-06-15 Por tôpico David M. Cardoso
L = nº laranjas P = nº peras X = nº pessoas Faça: 3L = P 5X = L 8X + 21 = P Serei a calculadora: -3L = -P 8x + 21 = P 8x - 3L = -21 5X - L = 0 (vezes -3) -15x +3L = 0 -7X = -21 == X = 3 == L = 15 == P = 45

Re: [obm-l] Problema envolvendo sistema linear

2004-06-15 Por tôpico Igor Castro
p - nº de peras l - nº de laranjas 3l = p sejam n amigos 5n = l 8n + 21 = p daí, 8n + 21 = 3l = 15n -7n=21 - n=3(3 amigos) ; l = 5n - 15 laranjas - Original Message - From: Maurizio [EMAIL PROTECTED] To: [EMAIL PROTECTED] Sent: Tuesday, June 15, 2004 7:16 PM Subject: [obm-l] Problema

Re: [obm-l] Problema envolvendo sistema linear

2004-06-15 Por tôpico Gloria Moura
Olá, Você tem: y = no. de laranjas x = no. de peras n = no. de pessoas x = 3 y y = 5 n 3 y - 8 n = 21 resolvendo: 15 n - 8 n = 21 ; n = 3 ; no. de laranjas = 5 n = 15 no. de peras = 3 y = 45 []s Glória Maurizio [EMAIL PROTECTED] wrote: Em uma cesta de frutas, há 3 vezes mais peras doq ue

RE: [obm-l] Problema 16 OBM - Nivel 3

2004-06-08 Por tôpico Rogério Moraes de Carvalho
= 2} Resposta: S = [1, 2] = {x real | 1 = x = 2} Abraços, Rogério Moraes de Carvalho -Original Message- From: [EMAIL PROTECTED] [mailto:[EMAIL PROTECTED] On Behalf Of Maurizio Sent: segunda-feira, 7 de junho de 2004 19:22 To: [EMAIL PROTECTED] Subject: [obm-l] Problema 16 OBM - Nivel 3

Re: RE: [obm-l] Problema 16 OBM - Nivel 3

2004-06-08 Por tôpico saulonpb
PROTECTED] [mailto:[EMAIL PROTECTED] On Behalf Of Maurizio Sent: segunda-feira, 7 de junho de 2004 19:22 To: [EMAIL PROTECTED] Subject: [obm-l] Problema 16 OBM - Nivel 3 Olá a questão 16 é assim: [x+2(x-1)^1/2]1/2+[x-2(x-1)^1/2]1/2=2 Eu obtive essa resoluçãoi mas não está dando certo... Quem

[obm-l] Problema 16 OBM - Nivel 3

2004-06-07 Por tôpico Maurizio
Olá a questão 16 é assim: [x+2(x-1)^1/2]1/2+[x-2(x-1)^1/2]1/2=2 Eu obtive essa resoluçãoi mas não está dando certo... Quem escrever alguma resolução ou indicar o erro da minha eu agradeço desde já {[x+2(x-1)^1/2]1/2}^2+2{[x+2(x-1)^1/2]1/2.[x-2(x-1)^1/2]1/2]}+{[x-2(x-1)^1/2]1/2}^2=4

Re: [obm-l] Problema 16 OBM - Nivel 3

2004-06-07 Por tôpico Igor Castro
PM Subject: [obm-l] Problema 16 OBM - Nivel 3 Olá a questão 16 é assim: [x+2(x-1)^1/2]1/2+[x-2(x-1)^1/2]1/2=2 Eu obtive essa resoluçãoi mas não está dando certo... Quem escrever alguma resolução ou indicar o erro da minha eu agradeço desde já {[x+2(x-1)^1/2]1/2}^2+2{[x+2(x-1)^1/2]1/2.[x-2(x

Re: [obm-l] Problema 16 OBM - Nivel 3

2004-06-07 Por tôpico Claudio Buffara
on 07.06.04 19:22, Maurizio at [EMAIL PROTECTED] wrote: Olá a questão 16 é assim: [x+2(x-1)^1/2]1/2+[x-2(x-1)^1/2]1/2=2 Eu obtive essa resoluçãoi mas não está dando certo... Quem escrever alguma resolução ou indicar o erro da minha eu agradeço desde já

[obm-l] Re: [obm-l] Problema dos canhões

2004-06-02 Por tôpico Rogerio Ponce
Olá pessoal, pela simetria, os mísseis permanecem formando um polígono regular de n lados. Como , a cada instante, um míssel viaja na direção do alvo com a velocidade de V, e este viaja nesta mesma direção (se afastando) com a velocidade de V * cos[360/n] , o encontro se dará em

[obm-l] Problema dos canhões

2004-06-01 Por tôpico Claudio Buffara
Title: Problema dos canhões Oi, pessoal: Há algum tempo o Wellington mandou o problema abaixo pra lista. Na época eu dei uma solução, mas hoje percebi que estava errada. Finalmente, após uma troca de msgs particulares, acho que ele e eu chegamos a um consenso. Mesmo assim, eu gostaria de ver

[obm-l] Problema de combinatória

2004-05-30 Por tôpico Fernando Villar
Olá pessoal, é um prazer participar desta lista. Resolvi o problema abaixo dividindo-o em muitos casos. "Quantos números de 3 algarismos distintos são divisíveis por 6?" Peço sugestõespara umaresolução mais suscinta. Agradeço

Re: [obm-l] Problema

2004-05-30 Por tôpico Johann Peter Gustav Lejeune Dirichlet
4 11:41:34 -0300 (ART) Subject: [obm-l] Problema Gostaria de saber se alguém poderia me ajudar com o seguinte problema: Sejam A e Banéis ordenados. Diz-se que umhomomorfismo injetivo f:A -- B preserva ordem se, para todoa 0 em A, tivermosf(a) ; 0. Sejam K um corpo ordenado e f:Q -- K um ho

Re: [obm-l] Problema

2004-05-29 Por tôpico Augusto Cesar de Oliveira Morgado
) 2295-3331        Fax: (21) 2295-2978 Empresa 100% Brasileira - Desde 1992 prestando servicos online -- Original Message --- From: Lista OBM [EMAIL PROTECTED] To: [EMAIL PROTECTED] Sent: Fri, 28 May 2004 11:41:34 -0300 (ART) Subject: [obm-l] Problema Gostaria de saber se

Re: [obm-l] Problema

2004-05-29 Por tôpico Lista OBM
--- From: Lista OBM [EMAIL PROTECTED] To: [EMAIL PROTECTED] Sent: Fri, 28 May 2004 11:41:34 -0300 (ART) Subject: [obm-l] Problema Gostaria de saber se alguém poderia me ajudar com o seguinte problema: Sejam A e Banéis ordenados. Diz-se que umhomomorfismo injetivo f:A -- B preserva ordem se, para

Re: [obm-l] Problema

2004-05-29 Por tôpico Augusto Cesar de Oliveira Morgado
) 2542-4849, (21) 2295-3331        Fax: (21) 2295-2978 Empresa 100% Brasileira - Desde 1992 prestando servicos online -- Original Message --- From: Lista OBM [EMAIL PROTECTED] To: [EMAIL PROTECTED] Sent: Sat, 29 May 2004 08:54:55 -0300 (ART) Subject: Re: [obm-l] Problema Meu

Re: [obm-l] Problema

2004-05-29 Por tôpico claudio.buffara
From: Lista OBM [EMAIL PROTECTED] To: [EMAIL PROTECTED] Sent: Fri, 28 May 2004 11:41:34 -0300 (ART) Subject: [obm-l] Problema Gostaria de saber se alguém poderia me ajudar com o seguinte problema: Sejam A e Banéis ordenados. Diz-se que umhomomorfismo injetivo f:A -- B preserva ordem se, para

Re: [obm-l] Problema

2004-05-29 Por tôpico Lista OBM
CTED] Sent: Fri, 28 May 2004 11:41:34 -0300 (ART) Subject: [obm-l] Problema Gostaria de saber se alguém poderia me ajudar com o seguinte problema: Sejam A e Banéis ordenados. Diz-se que umhomomorfismo injetivo f:A -- B preserva ordem se, para todoa 0 em A, tivermosf(a) ; 0. Sejam K um corpo orde

Re: [obm-l] Problema

2004-05-29 Por tôpico claudio.buffara
lema algum. []s, Claudio. De: [EMAIL PROTECTED] Para: [EMAIL PROTECTED] Cópia: Data: Sat, 29 May 2004 17:20:48 -0300 (ART) Assunto: Re: [obm-l] Problema Meu caro Cláudio, meu nome é Éder Franklin da Silva.Meu login é Lista OBM porque tenho mais de um e-mail no Yahoo (pr

Re: [obm-l] Problema

2004-05-29 Por tôpico Lista OBM
PROTECTED] Sent: Sat, 29 May 2004 08:54:55 -0300 (ART) Subject: Re: [obm-l] Problema Meu caro Morgado, não sei se estou equivocado, mas a aplicação que você definiu não é um homomorfismo, pois: f(x.y) = -(x.y) = -x.y e f(x).f(y) =(-x).(-y) = x.y, ou seja, f(x.y) é diferente de f(x).f(y). Além do

Re: [obm-l] Problema

2004-05-29 Por tôpico Lista OBM
Silva,por exemplo.Eu soh acho que seria meio esquisito comecar uma msg com "Caro Lista OBM", mas se voce preferir assim, nao vejo problema algum. []s, Claudio. De: [EMAIL PROTECTED] Para: [EMAIL PROTECTED] Cópia: Data: Sat, 29 May 2004 17:20:48 -0300 (ART) Assunto:

[obm-l] Problema F170b Kumon

2004-05-29 Por tôpico Maurizio
Problema do Kumon, meu prof pediu uma nova solução além da resolvível com lacunas... Plz ajudem! Obrigado Astrobaldo e Astrogilda estão repartindo 1600 ml de suco de uva. Astrogilda recebe 3/5 da parte que coube a Astrobaldo menos 40ml. Qtos cada um recebe?

<    6   7   8   9   10   11   12   13   14   15   >